Q1

 
skapur777
Thanks Received: 6
Atticus Finch
Atticus Finch
 
Posts: 145
Joined: March 27th, 2011
 
 
 

Q1

by skapur777 Thu Apr 14, 2011 12:20 am

I got this answer correct, but as usual, I'm trying to see why the others are incorrect (which explains my millions of post per night, it seems!). I was between A and C. Is C incorrect because the dichotomy is not illustrated in the novels themselves but in the various critics' widely divergent viewpoints? But the critics had to get their viewpoints for something, and clearly there is some middle ground that these novels are treading to elicit such opposing interpretations...perhaps then, the dichotomy is not well-illustrated because the lines are blurred in the author's novels?
 
giladedelman
Thanks Received: 833
LSAT Geek
 
Posts: 619
Joined: April 04th, 2010
 
This post thanked 1 time.
 
 

Re: Q1

by giladedelman Sun Apr 17, 2011 2:11 am

Good post! I agree that (C) is tempting. But there are a few things wrong with it.

First, you make a nice point about how if a work of literature is blurring the lines between something, that's kind of the opposite of illustrating a dichotomy.

(I'm actually not sure that even matters here, because the passage never addresses this particular dichotomy. Okay, maybe "highbrow" and "true literature" are close enough to "sophisticated," but where do we get "popular"? I don't think we do! We get "enjoyable" and "lowbrow," but never does the passage actually address whether that's popular.)

Ultimately, the crucial issue here, and the best reason to get rid of answer (C), is that we are being asked for the author's main conclusion. The author doesn't merely conclude that P.D. James's works illustrate something; no, the conclusion is much stronger, and is articulated as clearly as you could ask for in the final paragraph: "But perhaps the time has come for P. D. James to slide out of her handcuffs and stride into the territory of the mainstream novel." That's the author's conclusion, so an answer that doesn't mention this point can't possibly be correct here.

(A), on the other hand, neatly rehashes this conclusion, which is why it's correct.

(B) is out; the author never says popular novels and true creative expression are incompatible, and certainly doesn't think James's potential is diminished.

(D) is totally unsupported: we know the characters are painstakingly constructed, but not the plots. And even if this were true it would obviously be too narrow in scope.

(E) is the exact opposite of the author's conclusion.

Does that clear this up for you?
 
skapur777
Thanks Received: 6
Atticus Finch
Atticus Finch
 
Posts: 145
Joined: March 27th, 2011
 
 
 

Re: Q1

by skapur777 Mon Apr 18, 2011 10:15 pm

Yep! Thank you.